Difference between revisions of "2011 AIME I Problems/Problem 8"

m (Problem: Fixed graphic error.)
(Problem: ...Ok, now it)
Line 3: Line 3:
  
  
[asy]
+
<asy>
 
unitsize(1 cm);
 
unitsize(1 cm);
 
pair translate;
 
pair translate;
Line 34: Line 34:
 
draw (W[1]--C[1]--X[1]);
 
draw (W[1]--C[1]--X[1]);
 
draw (Y[1]--B[1]--Z[1]);
 
draw (Y[1]--B[1]--Z[1]);
dot("<math>A</math>",A[0],N);
+
dot("$A$",A[0],N);
dot("<math>B</math>",B[0],SE);
+
dot("$B$",B[0],SE);
dot("<math>C</math>",C[0],SW);
+
dot("$C$",C[0],SW);
dot("<math>U</math>",U[0],NE);
+
dot("$U$",U[0],NE);
dot("<math>V</math>",V[0],NW);
+
dot("$V$",V[0],NW);
dot("<math>W</math>",W[0],NW);
+
dot("$W$",W[0],NW);
dot("<math>X</math>",X[0],S);
+
dot("$X$",X[0],S);
dot("<math>Y</math>",Y[0],S);
+
dot("$Y$",Y[0],S);
dot("<math>Z</math>",Z[0],NE);
+
dot("$Z$",Z[0],NE);
 
dot(A[1]);
 
dot(A[1]);
 
dot(B[1]);
 
dot(B[1]);
 
dot(C[1]);
 
dot(C[1]);
dot("<math>U</math>",U[1],NE);
+
dot("$U$",U[1],NE);
dot("<math>V</math>",V[1],NW);
+
dot("$V$",V[1],NW);
dot("<math>W</math>",W[1],NW);
+
dot("$W$",W[1],NW);
dot("<math>X</math>",X[1],dir(-70));
+
dot("$X$",X[1],dir(-70));
dot("<math>Y</math>",Y[1],dir(250));
+
dot("$Y$",Y[1],dir(250));
dot("<math>Z</math>",Z[1],NE);[/asy]
+
dot("$Z$",Z[1],NE);</asy>
  
 
== See also ==
 
== See also ==
 
{{AIME box|year=2011|n=I|num-b=7|num-a=9}}
 
{{AIME box|year=2011|n=I|num-b=7|num-a=9}}

Revision as of 20:37, 3 April 2011

Problem

In triangle $ABC$, $BC = 23$, $CA = 27$, and $AB = 30$. Points $V$ and $W$ are on $\overline{AC}$ with $V$ on $\overline{AW}$, points $X$ and $Y$ are on $\overline{BC}$ with $X$ on $\overline{CY}$, and points $Z$ and $U$ are on $\overline{AB}$ with $Z$ on $\overline{BU}$. In addition, the points are positioned so that $\overline{UV}\parallel\overline{BC}$, $\overline{WX}\parallel\overline{AB}$, and $\overline{YZ}\parallel\overline{CA}$. Right angle folds are then made along $\overline{UV}$, $\overline{WX}$, and $\overline{YZ}$. The resulting figure is placed on a level floor to make a table with triangular legs. Let $h$ be the maximum possible height of a table constructed from triangle $ABC$ whose top is parallel to the floor. Then $h$ can be written in the form $\frac{k\sqrt{m}}{n}$, where $k$ and $n$ are relatively prime positive integers and $m$ is a positive integer that is not divisible by the square of any prime. Find $k+m+n$.


[asy] unitsize(1 cm); pair translate; pair[] A, B, C, U, V, W, X, Y, Z; A[0] = (1.5,2.8); B[0] = (3.2,0); C[0] = (0,0); U[0] = (0.69*A[0] + 0.31*B[0]); V[0] = (0.69*A[0] + 0.31*C[0]); W[0] = (0.69*C[0] + 0.31*A[0]); X[0] = (0.69*C[0] + 0.31*B[0]); Y[0] = (0.69*B[0] + 0.31*C[0]); Z[0] = (0.69*B[0] + 0.31*A[0]); translate = (7,0); A[1] = (1.3,1.1) + translate; B[1] = (2.4,-0.7) + translate; C[1] = (0.6,-0.7) + translate; U[1] = U[0] + translate; V[1] = V[0] + translate; W[1] = W[0] + translate; X[1] = X[0] + translate; Y[1] = Y[0] + translate; Z[1] = Z[0] + translate; draw (A[0]--B[0]--C[0]--cycle); draw (U[0]--V[0],dashed); draw (W[0]--X[0],dashed); draw (Y[0]--Z[0],dashed); draw (U[1]--V[1]--W[1]--X[1]--Y[1]--Z[1]--cycle); draw (U[1]--A[1]--V[1],dashed); draw (W[1]--C[1]--X[1]); draw (Y[1]--B[1]--Z[1]); dot("$A$",A[0],N); dot("$B$",B[0],SE); dot("$C$",C[0],SW); dot("$U$",U[0],NE); dot("$V$",V[0],NW); dot("$W$",W[0],NW); dot("$X$",X[0],S); dot("$Y$",Y[0],S); dot("$Z$",Z[0],NE); dot(A[1]); dot(B[1]); dot(C[1]); dot("$U$",U[1],NE); dot("$V$",V[1],NW); dot("$W$",W[1],NW); dot("$X$",X[1],dir(-70)); dot("$Y$",Y[1],dir(250)); dot("$Z$",Z[1],NE);[/asy]

See also

2011 AIME I (ProblemsAnswer KeyResources)
Preceded by
Problem 7
Followed by
Problem 9
1 2 3 4 5 6 7 8 9 10 11 12 13 14 15
All AIME Problems and Solutions